めもめも

このブログに記載の内容は個人の見解であり、必ずしも所属組織の立場、戦略、意見を代表するものではありません。

Quantum Information and Quantum Optics with Superconducting Circuits - Exercise Solutions (Chapter 9)

9.1

(1) \displaystyle U = e^{-i\frac{\theta(\epsilon)}{2}\sigma^y} = \cos\frac{\theta}{2}\,\mathbb 1 - i\sin\frac{\theta}{2}\,\sigma^y

 \displaystyle U\sigma_zU^\dagger = \left(\cos\frac{\theta}{2}\,\mathbb 1 - i\sin\frac{\theta}{2}\,\sigma^y\right)\sigma_z\left(\cos\frac{\theta}{2}\,\mathbb 1 + i\sin\frac{\theta}{2}\,\sigma^y\right)

  \displaystyle = \cos\theta\,\sigma_z + \sin\theta\,\sigma_x

So, by setting \displaystyle \tan\theta = \frac{\Delta_{\rm min}}{\epsilon}, we have:

 \displaystyle H = \frac{\epsilon}{2}\sigma^z + \frac{\Delta_{\rm min}}{2}\sigma^x = \frac{1}{2}\sqrt{\epsilon^2+\Delta_{\rm min}^2}(\cos\theta\,\sigma_z + \sin\theta\,\sigma_x) = \frac{1}{2}\Delta(\epsilon)U\sigma^zU^\dagger

where \displaystyle \Delta(\epsilon) := \sqrt{\epsilon^2+\Delta_{\rm min}^2}


(2) We apply an adiabatic change to \epsilon(t).

 \displaystyle \mid\Psi(t)\rangle = U(t)\mid\xi(t)\rangle

 \displaystyle H\mid\Psi(t)\rangle = \frac{1}{2}\Delta(\epsilon)U(t)\sigma^zU(t)^\dagger U(t)\mid\xi(t)\rangle = \frac{1}{2}\Delta(\epsilon)U(t)\sigma^z\mid\xi(t)\rangle

On the other hand,

 \displaystyle i\frac{d}{dt}\mid\Psi(t)\rangle =  i\frac{d}{dt}\left\{U(t)\mid\xi(t)\rangle\right\} = i\dot U(t)\mid\xi(t)\rangle + iU(t)\frac{d}{dt}\mid\xi(t)\rangle

Hence, from the Schroedinger equation \displaystyle H\mid\Psi(t)\rangle = i\frac{d}{dt}\mid\Psi(t)\rangle, we have:

 \displaystyle i\frac{d}{dt}\mid\xi(t)\rangle = -iU^\dagger(t)\dot U(t)\mid\xi(t)\rangle + \frac{1}{2}\Delta(\epsilon)\sigma^z\mid\xi(t)\rangle

From the definition of U, we have:

 \displaystyle\dot U(t) = -i\frac{\dot\theta(t)}{2}\sigma^y U(t)

So, \displaystyle -iU^\dagger(t)\dot U(t) = -\frac{\dot\theta(t)}{2}\sigma^y

 \displaystyle\therefore\, i\frac{d}{dt}\mid\xi(t)\rangle = -\frac{\dot\theta(t)}{2}\sigma^y\mid\xi(t)\rangle + \frac{1}{2}\Delta(\epsilon)\sigma^z\mid\xi(t)\rangle --- (2-1)

By differentiating the both sides of \displaystyle \epsilon(t)\tan\theta(t) = \Delta_{\rm min},

 \displaystyle \dot\epsilon(t)\tan\theta(t) + \epsilon(t)\frac{\dot\theta(t)}{\cos^2\theta(t)} = 0

 \displaystyle\therefore\, \dot\theta(t) = -\frac{\dot\epsilon(t)}{\epsilon(t)}\sin\theta\cos\theta = -\frac{\Delta_{\rm min}}{\Delta^2(\epsilon)}\dot\epsilon(t)

So, from (2-1), we have:

 \displaystyle\therefore\, i\frac{d}{dt}\mid\xi(t)\rangle = \left\{\frac{1}{2}\Delta(\epsilon)\sigma^z+\frac{\Delta_{\rm min}\dot\epsilon(t)}{\Delta^2(\epsilon)}\sigma^y\right\}\mid\xi(t)\rangle


(3) Since \displaystyle \frac{d}{dt} = \dot\epsilon(t)\frac{d}{d\epsilon}, we have:

 \displaystyle\therefore\, i\frac{d}{d\epsilon}\mid\xi(\epsilon)\rangle = \left\{\frac{1}{2\dot\epsilon}\Delta(\epsilon)\sigma^z+\frac{\Delta_{\rm min}}{\Delta^2(\epsilon)}\sigma^y\right\}\mid\xi(\epsilon)\rangle


(4) \displaystyle H_{\rm eff} = \frac{1}{\Delta}\sigma^z + \frac{\Delta_{\min}\dot\epsilon}{2\Delta^2}\sigma^y
=\frac{\Delta}{2}\begin{pmatrix}1 & -ig \\ ig & -1\end{pmatrix}

where \displaystyle g = \frac{\Delta_{\min}\dot\epsilon}{\Delta^3}

The eigenvalues of H_{\rm eff} are \displaystyle E_{\pm} = \pm\frac{\Delta}{2}\sqrt{1+g^2}

The eigenvector with E_- is given by:

 \displaystyle \mid 0 \rangle \propto \begin{pmatrix}\frac{i}{g}\left(-1\sqrt{1+g^2}\right) \\ 1 \end{pmatrix} = \begin{pmatrix}i\frac{g}{2} \\ 1\end{pmatrix}

Hence, the correction is \displaystyle \frac{g}{2} = \frac{\Delta_{\rm min}\dot\epsilon}{2\Delta^3}

9.2

The change should be slower than the spontaneous photon emission time of the qubit.

9.3

 \displaystyle H=\frac{1}{2}\sum_{i=1}^{N-1}J_i\sigma_i^z\sigma_{i+1}^z

Fix the direction of the 1st spin as \mid\psi_1\rangle = \mid 0\rangle, and decide the remaining spins with the following induction:

 \displaystyle  \mid \psi_{i+1}\rangle = -{\rm sign}(J_i)\mid\psi_i\rangle\ (i=2, 3,\cdots)

9.4

(1) We usually choose H_0 as a non-interacting system such as \displaystyle H_0 = \sum_i\Delta_i\sigma_i^x in (9.18) or (9.12). So the time evolution under H_0 is local.

(2) We need at least one gate for one interaction. So the number of gates is \sim dN^d\ (d=1,2,3).

9.5

The variance of sample average \overline x scales as \displaystyle \frac{1}{M}.

 \displaystyle V(\overline x) = V\left(\sum_{i=1}^M\frac{x_i}{M}\right) = \frac{1}{M^2}\sum_{i=1}^MV(x_i) = \frac{V(x)}{M}

So, the deviation scales as \displaystyle \sqrt{V(\overline x)} \sim \frac{1}{\sqrt{M}}

Since the Hamiltonian has N^2 terms, the total variance of the estimated Energy (as a sample average) scales as \displaystyle \frac{N^4}{M}. So, to keep the total variance below \epsilon, we have:

 \displaystyle \frac{N^4}{M} < \epsilon

 \displaystyle \therefore M > \frac{N^4}{\epsilon}

9.6

In the limit of \displaystyle |J_{\rm f}| \gg 1, we ignore other terms. Then the ground state is apparently the one where all spins have the same direction. Flipping one spin, we have the minimum gap \Delta_{\rm min} = 2|J_{\rm f}|.

As the initial state (the ground state of H_0 is a superposition of all states, it has an overlap with the ground state of \displaystyle |J_{\rm f}| \gg 1. So, with the adiabatic process (in the limit of \dot\epsilon\sim 0), it achieves the state where all spins have the same direction, that is, the state of qubit 1 is copied to the N intermediate qubits.

9.7

In this case, "Success in M experiments" means "Success at least one of M experiments". So the success probability is calculated as "1 - probability of fails in all M experiments." Hence, the success probability is:

  p = 1 - (1-p_{\rm ramp})^M

If p_{\rm ramp} = 0.99, we need M \ge 2 to achieve p=0.9999.

Practically, t_{\rm TTS} is defined as M t_{\rm ramp} where M is the number of experiments to achieve the required success probability.

Quantum Information and Quantum Optics with Superconducting Circuits - Exercise Solutions (Chapter 8)

8.1

 \displaystyle H\mid 0\rangle = \frac{1}{\sqrt{2}}\left(\mid 0\rangle +\mid 1\rangle\right)

 \displaystyle \therefore\,H^{\otimes N}\mid 00\cdots0\rangle = \frac{1}{2^{N/2}}\left(\mid 0\rangle +\mid 1\rangle\right)\otimes\left(\mid 0\rangle +\mid 1\rangle\right)\otimes\cdots\left(\mid 0\rangle +\mid 1\rangle\right)

By expanding the tensor product, we have the all possible binary numbers.

As discussed in 6.1.4, for a single qubit, Hadamard gate can be implemented as:

 \displaystyle H = e^{i\pi\sigma^z/2}e^{i\pi\sigma^y/2}

using the coherent drive:

 \displaystyle \epsilon(t) = \epsilon_0\cos(\omega_0t+\phi)

that's on resonance \omega_0=\Delta and \phi=\pi/2.

To apply it to N identical flux qubits, we have to have different gaps \Delta on neighboring qubits so that the drive doesn't affect neighboring qubits.

8.2

Since H\sigma^z H = \sigma^x and HH=1,

 (1\otimes H)U_{\rm CZ}(1\otimes H) = U_{\rm CNOT}

8.3

 \displaystyle H = \sum_{i=1}^2\left\{\frac{\Delta_i}{2}\sigma_i^z + g_i(\sigma_i^+\hat a + \hat a^\dagger\sigma_i^-)\right\}+\omega\hat a^\dagger \hat a

  \displaystyle =  \sum_{i=1}^2 \frac{\omega}{2}\sigma_i^z+\omega\hat a^\dagger \hat a + \sum_{i=1}^2\left\{\frac{\delta_i}{2}\sigma_i^z + g_i(\sigma_i^+\hat a + \hat a^\dagger\sigma_i^-)\right\}

where \delta_i := \Delta_i - \omega. Hence, we have:

 \displaystyle H = H_0 + H_1

 \displaystyle H_0 =\sum_{i=1}^2 \frac{\omega}{2}\sigma_i^z+\omega\hat a^\dagger \hat a

 \displaystyle H_1 =  \sum_{i=1}^2\left\{\frac{\delta_i}{2}\sigma_i^z + g_i(\sigma_i^+\hat a + \hat a^\dagger\sigma_i^-)\right\}

Then,

 \displaystyle [H_0,H_1] = \sum_{i,j}\frac{\omega g_j}{2}\left([\sigma_i^z,\sigma_j^+]\hat a + [\sigma_i^z,\sigma_j^-]\hat a^\dagger\right)

      \displaystyle {} + \sum_j\omega g_j\left(\sigma_j^+[\hat a^\dagger\hat a,\hat a] + \sigma_j^-[\hat a^\dagger\hat a,\hat a^\dagger]\right)

  \displaystyle = \sum_{i,j}\frac{\omega g_j}{2}\left(2\sigma_j^+\delta_{ij}\hat a - 2\sigma_j^-\delta_{ij}\hat a^\dagger\right) + \sum_j\omega g_j\left(-\sigma_j^+\hat a + \sigma_j^-\hat a^\dagger\right)

  \displaystyle = 0

So in the interaction picture with H_0, the system is described with H_1 itself. Note that, in this picture, the photon has no energy. (Photon energy is absorbed in \delta_i).

We split H_1 as:

 \displaystyle H_1 = \sum_{i=1}^2\frac{\delta_i}{2}\sigma_i^z + V

 \displaystyle V = \sum_{i=1}^2g_i(\sigma_i^+\hat a + \hat a^\dagger\sigma_i^-)

and take V as a weak perturbation.

Now we apply the nondegenerate perturbation theory. In general, the transformation generator  S_0 is given by:

 \displaystyle S_0 = \sum_{n\ne m}\frac{\langle n\mid V\mid m\rangle}{E_n-E_m}\mid n\rangle\langle m\mid

We consider the states with minimum number of photons (or N=2 subspace of JC-Ladder), such as:

 \mid 00\rangle\otimes\mid 2\rangle,\,\mid 10\rangle\otimes\mid 1\rangle,\,\mid 01\rangle\otimes\mid 1\rangle,\,\mid 11\rangle\otimes\mid 0\rangle

Since photons have no energy, their respective energy are:

 0,\,\delta_1,\,\delta_2,\,\delta_1 + \delta_2

Then, S_0 can be expanded as:

 \displaystyle S_0 = \frac{\sqrt{2}g_1}{\delta_1}\mid 10\rangle\otimes\mid 1\rangle\langle 00\mid\otimes\langle 2\mid +
 \frac{\sqrt{2}g_2}{\delta_2}\mid 01\rangle\otimes\mid 1\rangle\langle 00\mid\otimes\langle 2\mid

   \displaystyle {} + \frac{g_1}{\delta_1}\mid 11\rangle\otimes\mid 0\rangle\langle 01\mid\otimes\langle 1\mid +
 \frac{g_2}{\delta_2}\mid 11\rangle\otimes\mid 0\rangle\langle 10\mid\otimes\langle 1\mid
 
   \displaystyle {}- \frac{\sqrt{2}g_1}{\delta_1}\mid 00\rangle\otimes\mid 2\rangle\langle 10\mid\otimes\langle 1\mid -
 \frac{\sqrt{2}g_2}{\delta_2}\mid 00\rangle\otimes\mid 2\rangle\langle 01\mid\otimes\langle 2\mid

   \displaystyle {} - \frac{g_1}{\delta_1}\mid 01\rangle\otimes\mid 1\rangle\langle 11\mid\otimes\langle 0\mid +
 \frac{g_2}{\delta_2}\mid 10\rangle\otimes\mid 1\rangle\langle 11\mid\otimes\langle 0\mid

  \displaystyle = \sum_{i=1}^2\frac{g_i}{\delta_i}\left(\sigma_i^+\hat a - \sigma_i^-\hat a^\dagger\right)

Note that, with similar calculations, you can confirm that the above expression for S_0 is valid for any subspace of JC-Ladder though we explicitly calculated in N=2 subspace.

So the perturbative correction to the Hamiltonian is given by:

 \displaystyle \frac{1}{2}[S_0,V] = \frac{1}{2}\sum_{i,j}\frac{g_ig_j}{\delta_i}\left([\sigma^+_i\hat a,\sigma_j^-\hat a^\dagger]-[\sigma_i^-\hat a^\dagger,\sigma_j^+\hat a]\right)

  \displaystyle = \frac{1}{2}\sum_{i,j}\left\{\frac{g_ig_j}{\delta_i}[\sigma^+_i\hat a,\sigma_j^-\hat a^\dagger] + (i \leftrightarrow j)\right\}

Now,

 \displaystyle [\sigma^+_i\hat a,\sigma_j^-\hat a^\dagger] = \sigma_i^+\sigma_j^-\hat a\hat a^\dagger - \sigma_j^-\sigma_i^+\hat a^\dagger\hat a

  \displaystyle = \sigma_i^+\sigma_j^-(\hat a^\dagger\hat a + 1) - \sigma_j^-\sigma_i^+\hat a^\dagger\hat a

  \displaystyle = [\sigma_i^+,\sigma_j^-]\hat a^\dagger\hat a + \sigma_i^+\sigma_j^-

  \displaystyle = \delta_{ij}\sigma_i^z\hat a^\dagger\hat a + \sigma_i^+\sigma_j^-

Hence,

 \displaystyle  \frac{1}{2}[S_0,V] = \frac{1}{2}\sum_{i,j}\left\{\frac{g_ig_j}{\delta_i}\left(\delta_{ij}\sigma_i^z\hat a^\dagger\hat a + \sigma_i^+\sigma_j^-\right) + (i \leftrightarrow j)\right\}

  \displaystyle = \sum_i\frac{g_i^2}{\delta_i}\sigma_i^z\hat a^\dagger\hat a + \sum_i\frac{g_i^2}{\delta_i}\sigma_i^+\sigma_i^- + \frac{1}{2}\sum_{i\ne j}\frac{g_ig_j}{\delta_i}(\sigma_i^+\sigma_j^-+\sigma_j^+\sigma_i^-)

  \displaystyle = \sum_i\frac{g_i^2}{\delta_i}\sigma_i^z\hat a^\dagger\hat a + \frac{1}{2}\sum_i\frac{g_i^2}{\delta_i}(1+\sigma_i^z) + \frac{g_1g_2}{2}\left(\frac{1}{\delta_1}+\frac{1}{\delta_2}\right)(\sigma_1^+\sigma_2^-+\sigma_1^-\sigma_2^+)

So, the effective Hamiltonian (in the interaction picture) is:

 \displaystyle H_{\rm eff} = \sum_{i=1}^2\frac{1}{2}\left(\delta_i + \frac{g_i^2}{\delta_i}\right)\sigma_i^z + \frac{g_1g_2}{2}\left(\frac{1}{\delta_1}+\frac{1}{\delta_2}\right)(\sigma_1^+\sigma_2^-+\sigma_1^-\sigma_2^+)

   \displaystyle {} + \sum_{i=1}^2\frac{g_i^2}{\delta_i}\sigma_i^z\hat a^\dagger\hat a + {\rm Const.} --- (1-1)

Since \displaystyle [H_0,S_0] =0, we can go back to the original Schroedinger picture without modifying H_0. So getting back to the original Schroedinger picture and dropping the photon energy \propto \hat a^\dagger\hat a, we have:

 \displaystyle H_{\rm eff} = \sum_{i=1}^2\frac{1}{2}\left(\Delta_i + \frac{g_i^2}{\delta_i}\right)\sigma_i^z + \frac{g_1g_2}{2}\left(\frac{1}{\delta_1}+\frac{1}{\delta_2}\right)(\sigma_1^+\sigma_2^-+\sigma_1^-\sigma_2^+)


(2) Defining \displaystyle\Delta'_i := \Delta_i +\frac{g_i^2}{\delta_i}, J :=  \frac{g_1g_2}{2}\left(\frac{1}{\delta_1}+\frac{1}{\delta_2}\right), we have:

 \displaystyle H_{\rm eff} = \sum_{i=1}^2\frac{\Delta'_i}{2}\sigma_i^z + J(\sigma_1^+\sigma_2^-+\sigma_1^-\sigma_2^+)

  \displaystyle=\begin{pmatrix}\frac{\Delta'_1+\Delta'_2}{2} & 0 & 0 & 0 \\
0 & \frac{\Delta'_1-\Delta'_2}{2} & J & 0 \\
0 & J & -\frac{\Delta'_1-\Delta'_2}{2} & 0 \\
0 & 0 & 0 & -\frac{\Delta'_1+\Delta'_2}{2}\end{pmatrix}

Hence states \mid 00\rangle, \,\mid 11\rangle are eigenstates of H_{\rm eff} whereas states \mid 00\rangle, \,\mid 11\rangle are mixed together via time-evolution.

As discussed in 8.3.2, when \Delta'_1 = \Delta'_2 =: \Delta', the time evolution operator becomes:

 \displaystyle U(t) = e^{-iH_{\rm eff}t} = e^{-i\Delta'(\sigma_1^z+\sigma_2^z)}\begin{pmatrix}
1 & 0 & 0 & 0\\
0 & \cos(Jt) & -i\sin(Jt) & 0 \\
0 & -i\sin(Jt) & \cos(Jt) & 0 \\
0 & 0 & 0 & 1\end{pmatrix}

So you can implement iSwap with \displaystyle U(3\pi / 2J) up to local transformations:

 \displaystyle U_{\rm iSWAP} = e^{-i\pi\Delta'(\sigma_1^z+\sigma_2^z)}U(3\pi/2J) --- (2-1)


(3) \displaystyle \langle \hat a^\dagger\hat a\rangle = \frac{1}{e^{\hbar\omega/kT}-1} \sim 0.67

From (1-1), the effective energy of qubits \Delta' increase \displaystyle \frac{2g_i^2}{\delta_i}\langle\hat a^\dagger\hat a\rangle \sim \frac{g_i^2}{\delta_i}. This would influence the local transformation in (2-1). But I don't see how it affects the two-qubit exchange???

8.4

We use the unit system with \hbar = 1, and a consistent notation for energy levels \omega_1,\,\omega_2 (instead of \Delta_1,\,\Delta_2).

(1) The eigenvalues of \mid 01\rangle,\,\mid 10\rangle subspace are:

 \displaystyle E_1,\,E_2 = \frac{\omega_1+\omega_2 - \sqrt{D_1}}{2},\,  \frac{\omega_1+\omega_2 + \sqrt{D_1}}{2}

where

 \displaystyle D_1 = (\omega_2-\omega_1)^2 + 4J^2

The eigenvalues of \mid 11\rangle,\,\mid 02\rangle subspace are:

 \displaystyle E_3,\,E_4 = \frac{-\alpha + \omega_1+3\omega_2 - \sqrt{D_2}}{2},\,  \frac{-\alpha + \omega_1+3\omega_2 + \sqrt{D_2}}{2}

where

 \displaystyle D_2 = (\omega_2-\omega_1-\alpha)^2 + 8J^2

gist.github.com


(2) From the results of (1), energy levels degenerate at D_1 = 0 and D_2=0, that is, \omega_1=\omega_2 and \omega_2 = \omega_1 + \alpha


(3) In D_1,\, D_2, for large \omega_2, we can ignore J. So approximating as J\sim 0, we have:

 E_1,\,E_2,\,E_3,\,E_4 = \omega_1,\,\omega_2,\,\omega_1+\omega_2,\,2\omega_2-\alpha

They respectively correspond to states \mid 10\rangle,\,\mid 01\rangle,\,\mid 11\rangle,\,\mid 02\rangle.


(4) Relabel the eigenvalues as:

 E_{10} := E_1,\,E_{01} := E_2,\,E_{11} := E_3,\,E_{02} := E_4

Then, the phase changes are described as:

 \displaystyle\theta(s_1,s_2) = -\int_0^TE_{s_1s_2}(t)\,dt


(5) Define:

 \displaystyle\Delta E(t) := \frac{1}{4}\left[\left\{E_{11}(t) + E_{00}(t)\right\} - \left\{E_{01}(t) + E_{10}(t)\right\}\right]

Then,

 \displaystyle\Delta\theta = \frac{1}{4}\left[\left\{\theta(1,1) + \theta(0,0)\right\} - \left\{\theta(0,1) + \theta(1,0)\right\}\right]

  \displaystyle = -\int_0^T\Delta E(t)\,dt

If we modify the adiabatic change n times slower, \Delta\theta is modified as:

 \displaystyle\Delta\theta' = -\int_0^{nT}\Delta E\left(\frac{t}{n}\right)\,dt = -n\int_0^T\Delta E(t)\,dt = n \Delta\theta

So, by adjusting the speed of the adiabatic change, we can achieve arbitrary value of \Delta\theta.

Note that once we achieve the transformation with \Delta\theta, by applying single qubit operations, we can achieve the transformation:

 \displaystyle U = \begin{pmatrix} 1 & 0 & 0 & 0 \\ 0 & 1 & 0 & 0 \\ 0 & 0 & 1 & 0 \\ 0 & 0 & 0 & e^{i\Delta\theta}\end{pmatrix}


(6) N-bit Quantum Fourier transformation can be implemented with control-phase gates:

 \displaystyle U(k) = \begin{pmatrix} 1 & 0 & 0 & 0 \\ 0 & 1 & 0 & 0 \\ 0 & 0 & 1 & 0 \\ 0 & 0 & 0 & e^{2\pi i/2^k}\end{pmatrix}\ (k=2,\cdots,N)

So we need \displaystyle\Delta\theta = \frac{2\pi}{2^k}.

8.5

 \displaystyle U(t) =\exp\left(-it\sigma_1^z\sigma_2^z\right) =
\begin{pmatrix} e^{-it} & 0 & 0 & 0 \\ 0 & e^{it} & 0 & 0 \\ 0 & 0 & e^{it} & 0 \\ 0 & 0 & 0 & e^{-it}\end{pmatrix}
=  \begin{pmatrix} e^{-i\sigma_z} & \mathbf 0 \\ \mathbf 0 & e^{i\sigma_z} \end{pmatrix}

 \displaystyle U_{\rm C-NOT} = \begin{pmatrix} 1 & 0 & 0 & 0 \\ 0 & 1 & 0 & 0 \\ 0 & 0 & 0 & 1 \\ 0 & 0 & 1 & 0\end{pmatrix}
=\begin{pmatrix} {\mathbf 1} & \mathbf 0 \\ \mathbf 0 & \sigma_x \end{pmatrix}

 \displaystyle U_{\rm C-NOT} (\mathbf 1\otimes \sigma_z) U_{\rm C-NOT} = 
\begin{pmatrix} {\mathbf 1} & \mathbf 0 \\ \mathbf 0 & \sigma_x \end{pmatrix}
\begin{pmatrix} \sigma_z & \mathbf 0 \\ \mathbf 0 & \sigma_z \end{pmatrix}
\begin{pmatrix} {\mathbf 1} & \mathbf 0 \\ \mathbf 0 & \sigma_x \end{pmatrix}

  \displaystyle = \begin{pmatrix} \sigma_z & \mathbf 0 \\ \mathbf 0 & \sigma_x\sigma_z\sigma_x \end{pmatrix}
=  \begin{pmatrix} \sigma_z & \mathbf 0 \\ \mathbf 0 & -\sigma_z \end{pmatrix}

Hence,

 \displaystyle U_{\rm C-NOT} (\mathbf 1\otimes e^{-i\sigma_z}) U_{\rm C-NOT} = \begin{pmatrix} e^{-i\sigma_z} & \mathbf 0 \\ \mathbf 0 & e^{i\sigma_z} \end{pmatrix} = U(t)

8.6

(1) Asymmetric totally depolarizing channel:

 \displaystyle \epsilon(\rho) = \left(1-\sum_{\alpha=x,y,z}p_\alpha\right)\rho + \sum_{\alpha=x,y,z}p_\alpha\sigma^\alpha\rho\sigma^\alpha

This applies the phase flip (along the \alpha axe) \sigma^\alpha with probability p_\alpha.

Since \displaystyle \frac{1}{2}\left(\rho + \sum_{\alpha=x,y,z}\sigma^\alpha\rho\sigma^\alpha\right) = \mathbb 1, by setting \displaystyle p_x=p_y=p_z=\frac{p}{4}, we have:

 \displaystyle\epsilon(\rho) = \left(1-\frac{3p}{4}\right)\rho + \frac{p}{4}\sum_{\alpha=x,y,z}\sigma^\alpha\rho\sigma^\alpha

  \displaystyle =  \left(1-p\right)\rho + \frac{p}{4}\left(\rho+\sum_{\alpha=x,y,z}\sigma^\alpha\rho\sigma^\alpha\right)

  \displaystyle = (1-p)\rho + p\frac{\mathbb 1}{2}

In this symmetric case, with probability p, the system is replaced with the completely mixed status \displaystyle \frac{\mathbb 1}{2}.


(2) Random flip channel is implemented as:

 \displaystyle \epsilon(\rho) = (1-p)\rho + p\sigma_x\rho\sigma_x

This is a special case of the asymmetric totally depolarizing channel p_x = p, p_y = p_z =0.


(3) For \displaystyle \rho = \begin{pmatrix} a & b \\ c & d\end{pmatrix},

 \displaystyle \epsilon(\rho) = \begin{pmatrix} a & (1-p)b \\ (1-p)c & d\end{pmatrix}

So applying it N times, we have:

  \displaystyle \epsilon^N(\rho) = \begin{pmatrix} a & (1-p)^Nb \\ (1-p)^Nc & d\end{pmatrix}


(4) Kraus operators for asymmetric totally depolarizing channel are:

 \displaystyle A_0 = \sqrt{1-\sum_{\alpha=x,y,z}p_\alpha}\,\mathbb 1,\, A_\alpha = \sqrt{p_\alpha}\sigma^\alpha\ (\alpha=x,y,z)

Kraus operators for dephasing channel are:

 \displaystyle A_0 = \sqrt{\frac{1}{2}(1+p)}\,\mathbb 1,\,A_1 = \sqrt{\frac{1}{2}(1-p)}\sigma^z

Actually,

 \displaystyle A_0\rho A_0^\dagger + A_1\rho A_1^\dagger = \frac{1}{2}(1+p)\begin{pmatrix} a & b \\ c & d\end{pmatrix} + \frac{1}{2}(1-p)\begin{pmatrix} a & -b \\ -c & d\end{pmatrix}

  \displaystyle = \begin{pmatrix} a & 0 \\ 0 & d\end{pmatrix} + p\begin{pmatrix} 0 & b \\ c & 0\end{pmatrix}
=p\begin{pmatrix} a & b \\ c & d\end{pmatrix} + (1-p)\begin{pmatrix} a & 0 \\ 0 & d\end{pmatrix}

Since {\rm tr}(\sigma^\alpha\rho\sigma^\alpha) = {\rm tr}\rho, for asymmetric totally depolarizing channel,

 \displaystyle {\rm tr}\epsilon(\rho) = \left(1-\sum p_\alpha\right){\rm tr}\rho + \sum p_\alpha\, {\rm tr}\rho = {\rm tr}\rho

For other channels, you can confirm \displaystyle {\rm tr}\epsilon(\rho) = {\rm tr}\rho with similar calculations.

8.7

Define \displaystyle \sigma_0 = 1 and you can confirm with a direct calculation that:

 \displaystyle {\rm tr}(\sigma^\dagger_{\alpha}\sigma_{\beta})=\delta_{\alpha\beta}\ (\alpha,\beta=x, y, z, 0).

SInce [tex|displaystyle {\rm tr}(A\otimes B)={\rm tr}(A){\rm tr}(B)], it's also true that:

 \displaystyle {\rm tr}(S^\dagger_{\alpha}S_{\beta})=\delta_{\alpha\beta}\ (S_\alpha,S_\beta \in \{1,\sigma^x,\sigma^y,\sigma^z\}^{\otimes N}).

So, under the inner product \displaystyle (S_\alpha,S_\beta) := {\rm tr}(S_\alpha^\dagger S_\beta) they are linear-independent and become orthobormal basis of 2^{2N} dimension linear space.

As the 2^N\times 2^N matrix \rho has 2^{2N} elements, it can be expressed by a liner combination of S_\alpha.

 \rho = \sum s_\alpha S^\alpha

And the coefficients are determined by \alpha = (\rho, S_\alpha).

8.8

In the textbook, matrix representation and operator notation are somewhat mixed up. In the operator notation, we have:

 \displaystyle \epsilon(\hat\rho) = \frac{1}{d}\sum_{\alpha,\beta}\sum_{i,j}M_{\beta\alpha}{\rm tr}(\hat S^\dagger_\alpha\hat\rho)(\hat S_\beta)_{ij}\mid i\rangle\langle j\mid --- (1)

where \displaystyle (\hat S_\beta)_{ij} := \langle i \mid\hat S_\beta\mid j \rangle

 \displaystyle \epsilon(\hat \rho) = \sum_{\alpha,\beta}\sum_{i,j}\chi_{\alpha\beta}(\hat S_\alpha\hat\rho\hat S^\dagger_\beta)_{ij}\mid i\rangle\langle j\mid --- (2)

where \displaystyle (\hat S_\alpha\hat\rho\hat S^\dagger_\beta)_{ij} := \langle i \mid\hat S_\alpha\hat\rho\hat S^\dagger_\beta\mid j \rangle

From (1), we have:

 \displaystyle \rho_\epsilon := (\mathbb 1\otimes\epsilon)\left(\frac{1}{d}\sum_{i,j}\mid i\rangle\langle j\mid\otimes\mid i\rangle\langle j\mid\right)= \frac{1}{d}\sum_{i,j}\mid i\rangle\langle j\mid\otimes\epsilon\left(\mid i\rangle\langle j\mid\right)

  \displaystyle = \frac{1}{d^2}\sum_{i,j}\mid i\rangle\langle j\mid\otimes\sum_{\alpha,\beta}\sum_{k,l}M_{\beta\alpha}{\rm tr}(\hat S^\dagger_\alpha\mid i\rangle\langle j\mid)(\hat S_\beta)_{kl}\mid k\rangle\langle l\mid

  \displaystyle = \frac{1}{d^2}\sum_{\alpha,\beta}\sum_{k,l}\sum_{i,j}\mid i\rangle\langle j\mid\otimes M_{\beta\alpha}(\hat S_\alpha)_{ji}(\hat S_\beta)_{kl}\mid k\rangle\langle l\mid

  \displaystyle = \frac{1}{d^2}\sum_{\alpha,\beta}\sum_{k,l}\sum_{i,j} M_{\beta\alpha}(\hat S_\alpha)^{\rm T}_{ij}\mid i\rangle\langle j\mid\otimes(\hat S_\beta)_{kl}\mid k\rangle\langle l\mid

  \displaystyle = \frac{1}{d^2}\sum_{\alpha,\beta}M_{\beta\alpha}\hat S^{\rm T}_\alpha\otimes \hat S_\beta.

From (2), we have:

 \displaystyle \rho_\epsilon := (\mathbb 1\otimes\epsilon)\left(\frac{1}{d}\sum_{i,j}\mid i\rangle\langle j\mid\otimes\mid i\rangle\langle j\mid\right)= \frac{1}{d}\sum_{i,j}\mid i\rangle\langle j\mid\otimes\epsilon\left(\mid i\rangle\langle j\mid\right)

  \displaystyle = \frac{1}{d}\sum_{i,j}\mid i\rangle\langle j\mid\otimes \sum_{\alpha,\beta}\sum_{k,l}\chi_{\alpha\beta}(\hat S_\alpha\mid i\rangle\langle j\mid \hat S^\dagger_\beta)_{kl}\mid k\rangle\langle l\mid

  \displaystyle = \frac{1}{d}\sum_{\alpha,\beta}\sum_{i,j}\chi_{\alpha\beta}\mid i\rangle\langle j\mid\otimes \hat S_\alpha\mid i\rangle\langle j\mid \hat S^\dagger_\beta

  \displaystyle = \frac{1}{d}\sum_{\alpha,\beta}\sum_{i,j}\chi_{\alpha\beta}\mid i\rangle\otimes\hat S_\alpha\mid i\rangle\langle j\mid\otimes\langle j\mid\hat S^\dagger_\beta

  \displaystyle = \sum_{\alpha,\beta}\chi_{\alpha\beta}\mid\alpha\rangle\langle\beta\mid

Now we prove (8.16). For any \hat\rho,

 \displaystyle\epsilon(\rho) = \frac{1}{d}\sum_{\alpha,\beta}M_{\beta\alpha}{\rm tr}(\hat S_\alpha\hat\rho)\hat S_\beta = \sum_{\alpha,\beta}\chi_{\alpha\beta}\hat S_\alpha\hat\rho\hat S_\beta^\dagger

On both sides, we apply \displaystyle {\rm tr}(\hat S_{\beta'}*) from left:

 \displaystyle \frac{1}{d}\sum_{\alpha,\beta}M_{\beta\alpha}{\rm tr}(\hat S_\alpha\hat\rho){\rm tr}(\hat S_{\beta'}\hat S_\beta) = \sum_{\alpha,\beta}\chi_{\alpha\beta}{\rm tr}(\hat S_{\beta'}\hat S_\alpha\hat\rho\hat S_\beta^\dagger)

Since \displaystyle {\rm tr}(\hat S_{\beta'}\hat S_\beta) = d\delta_{\beta\beta'}, we have:

 \displaystyle \sum_{\alpha}M_{\beta'\alpha}{\rm tr}(\hat S_\alpha\hat\rho) = \sum_{\alpha,\beta}\chi_{\alpha\beta}{\rm tr}(\hat S_{\beta'}\hat S_\alpha\hat\rho\hat S_\beta^\dagger) --- (3)

Now we take \displaystyle \hat\rho = \hat S_{\alpha'}, and using the relationship \displaystyle {\rm tr}(\hat S_\alpha\hat S_{\alpha'}) = d\delta_{\alpha\alpha'}, we have:

 \displaystyle M_{\beta'\alpha'} = \frac{1}{d} \sum_{\alpha,\beta}\chi_{\alpha\beta}{\rm tr}(\hat S_{\beta'}\hat S_\alpha\hat S_{\alpha'}\hat S_\beta^\dagger)

8.9

(1) Deduce M and \chi for \displaystyle \epsilon(\hat\rho) = U\hat\rho U^\dagger.

For M_{\alpha\beta}, from Exercise 8.8 (1), we have:

 \displaystyle \frac{1}{d}\sum_{\alpha,\beta}M_{\beta\alpha}{\rm tr}(\hat S^\dagger_\alpha\hat \rho)\hat S_\beta = U\hat\rho U^\dagger

Applying \displaystyle{\rm tr}(\hat S_{\beta'}\, *) and taking \displaystyle \hat\rho = \hat S_{\alpha'}, we have:

 \displaystyle d\sum_{\alpha,\beta}M_{\beta\alpha}\delta_{\alpha\alpha'}\delta_{\beta\beta'} = {\rm tr}(\hat S_{\beta'}U\hat S_{\alpha'} U^\dagger)

 \displaystyle\therefore\, M_{\beta\alpha} = \frac{1}{d}{\rm tr}(\hat S_\beta U\hat S_\alpha U^\dagger)

 \displaystyle\therefore\, M_{\alpha\beta} = \frac{1}{d}{\rm tr}(U^\dagger\hat S_\alpha U\hat S_\beta )

For \chi_{\alpha\beta},

 \displaystyle \mid\alpha \rangle = \frac{1}{\sqrt{d}}\sum_i\mid i\rangle\otimes \hat S_\alpha\mid i\rangle

 \displaystyle \langle\alpha\mid\beta\rangle = \frac{1}{d}\sum_{i,j}\langle i\mid j\rangle\otimes\langle i\mid \hat S^\dagger_\alpha \hat S_\beta\mid j\rangle= \frac{1}{d}{\rm tr}(\hat S_\alpha^\dagger \hat S_\beta) = \delta_{\alpha\beta}

Hence, from Exercise 8.8 (3),

 \displaystyle\chi_{\alpha\beta} = \langle\alpha\mid(\mathbb 1\otimes\epsilon)(\mid\Psi\rangle\langle\Psi\mid)\mid\beta\rangle

  \displaystyle = \frac{1}{d}\sum_{i,j}\langle\alpha\mid i\rangle\langle j\mid \otimes\epsilon(\mid i\rangle\langle j\mid)\mid\beta\rangle

  \displaystyle = \frac{1}{d}\sum_{i,j}\langle\alpha\mid i\rangle\langle j\mid \otimes U\mid i\rangle\langle j\mid U^\dagger \mid\beta\rangle

  \displaystyle = \frac{1}{d^2}\sum_{i,j,k,l}\langle k\mid i\rangle\langle j\mid l \rangle \otimes\langle k\mid \hat S^\dagger_\alpha U\mid i\rangle\langle j\mid U^\dagger\hat S_\beta\mid l\rangle

  \displaystyle = \frac{1}{d^2}{\rm tr}(\hat S^\dagger_\alpha U){\rm tr}(U^\dagger\hat S_\beta)


(2) M and \chi for \displaystyle U=\exp\left(-i\theta\mathbf n\cdot\mathbf\sigma\right).

We use the Einstein summation convention in the following calculations.

For M_{\alpha\beta},

 \displaystyle M_{\alpha\beta} = \frac{1}{d}{\rm tr}(U^\dagger \sigma_\alpha U\sigma_\beta )\ \ where \sigma_\alpha \in \{\mathbb 1, \sigma_x,\sigma_y,\sigma_z\}

If either \sigma_\alpha or \sigma_\beta is \mathbb 1,  M_{\alpha\beta} = 0.

For other cases, from the relationships:

 \displaystyle[\sigma_i,\sigma_j] = 2i\epsilon_{ijk}\sigma_k

 \displaystyle\sigma_i\sigma_j = \delta_{ij}\mathbb 1 + i\epsilon_{ijk}\sigma_k

 \displaystyle \exp(i\theta\mathbf n\cdot\mathbf\sigma) = (\cos\theta)\mathbb 1 + (i\sin\theta)\mathbf n\cdot\mathbf \sigma

we have:

 \displaystyle{\rm tr}(U^\dagger\sigma_i U\sigma_j) = {\rm tr}\left\{\exp(i\theta\mathbf n\cdot\mathbf\sigma)\sigma_i \exp(-i\theta\mathbf n\cdot\mathbf\sigma)\sigma_j\right\}

  \displaystyle = {\rm tr}\left[\left\{(\cos\theta)\mathbb 1 + (i\sin\theta)n_k\sigma_k\right\}\sigma_i
\left\{(\cos\theta)\mathbb 1 - (i\sin\theta)n_l\sigma_l\right\}\sigma_j\right]

  \displaystyle = \cos^2\theta\, {\rm tr}(\sigma_i\sigma_j)

   \displaystyle {} + i\sin\theta\cos\theta\,n_k{\rm tr}(\sigma_k\sigma_i\sigma_j) - i\sin\theta\cos\theta\,n_l{\rm tr}(\sigma_i\sigma_l\sigma_j)

   \displaystyle {} + \sin^2\theta\,n_kn_l\,{\rm tr}(\sigma_k\sigma_i\sigma_l\sigma_j)

  \displaystyle = 2\cos^2\theta\,\delta_{ij} + i\sin\theta\cos\theta\,n_k{\rm tr}(\sigma_k[\sigma_i, \sigma_j]) + \sin^2\theta\,n_kn_l\,{\rm tr}(\sigma_k\sigma_i\sigma_l\sigma_j)

 (1st term) \displaystyle = 2\cos^2\theta\,\mathbf e_i\cdot\mathbf e_j

 (2nd term) \displaystyle = -2\sin\theta\cos\theta\,n_k\,{\rm tr}(\sigma_k\epsilon_{ijl}\sigma_l) = -4 \sin\theta\cos\theta\,n_k\epsilon_{ijl}\delta_{kl}

  \displaystyle = -4\sin\theta\cos\theta \, (\mathbf e_i\times\mathbf e_j)\cdot\mathbf n

 (3rd term) \displaystyle = \sin^2\theta\,n_kn_l\,{\rm tr}\left\{
(\delta_{ki}\mathbb 1 + i\epsilon_{kim}\sigma_m)(\delta_{lj}\mathbb 1+i\epsilon_{ljn}\sigma_n)\right\}

  \displaystyle = 2 \sin^2\theta\,n_kn_l\,(\delta_{ki}\delta_{lj}-\epsilon_{kim}\epsilon_{ljm})

  \displaystyle = 2 \sin^2\theta\,n_kn_l\,\left\{\delta_{ki}\delta_{lj}-(\delta_{kl}\delta_{ij}-\delta_{kj}\delta_{il})\right\}

  \displaystyle = 2 \sin^2\theta\,(n_in_j - \delta_{ij} + n_jn_i)

  \displaystyle = 4\sin^2\theta\,(\mathbf e_i\cdot\mathbf n)(\mathbf e_j\cdot\mathbf n)-2\sin^2\theta\,\mathbf e_i\cdot\mathbf e_j

 \displaystyle\therefore  M_{ij} = \frac{1}{2}{\rm tr}(U^\dagger \sigma_i U\sigma_j)

  \displaystyle = (\cos^2\theta-\sin^2\theta)\,(\mathbf e_i\cdot\mathbf e_j) - 2\sin\theta\cos\theta \, (\mathbf e_i\times\mathbf e_j)\cdot\mathbf n + 2\sin^2\theta\,(\mathbf e_i\cdot\mathbf n)(\mathbf e_j\cdot\mathbf n)

For \chi_{\alpha\beta},

 \displaystyle \chi_{\alpha\beta}= \frac{1}{4}{\rm tr}(\sigma_\alpha U){\rm tr}(U^\dagger\sigma_\beta)\ \ where \sigma_\alpha \in \{\mathbb 1, \sigma_x,\sigma_y,\sigma_z\}

We use the convention that n_0 = 0, then we have:

 \displaystyle \chi_{\alpha\beta}= \frac{1}{4}{\rm tr}\left\{\sigma_\alpha \exp(-i\theta\mathbf n\cdot\mathbf\sigma)\right\}{\rm tr}\left\{\exp(i\theta\mathbf n\cdot\mathbf\sigma)\sigma_\beta\right\}

  \displaystyle =  \frac{1}{4}{\rm tr}\left[ \sigma_\alpha \left\{(\cos\theta)\mathbb 1 - (i\sin\theta)n_k \sigma_k\right\}\right]
{\rm tr}\left[ \left\{(\cos\theta)\mathbb 1 + (i\sin\theta)n_l \sigma_l\right\}\sigma_\beta \right]

  \displaystyle = \left(\cos\theta\,\delta_{\alpha 0}-i\sin\theta\,n_k\delta_{\alpha k}\right)
 \left(\cos\theta\,\delta_{\beta 0}+i\sin\theta\,n_l\delta_{\beta l}\right)

  \displaystyle = \cos^2\theta\,\delta_{\alpha 0}\delta_{\beta 0}+i\sin\theta\,n_\beta\delta_{\alpha 0}-i\sin\theta\,n_\alpha\delta_{\beta 0} + \sin^2\theta\, n_\alpha n_\beta


(3) M and \chi for CNOT \displaystyle U=U^\dagger = \,\mid 0\rangle\langle 0\mid \otimes \mathbb 1 + \mid 1\rangle\langle 1\mid\otimes\sigma_x.

 \displaystyle U(\sigma_\alpha\otimes\sigma_\beta) = \mid 0\rangle\langle 0\mid \sigma_\alpha\otimes\sigma_\beta + \mid 1\rangle\langle 1\mid\sigma_\alpha\otimes\sigma_x\sigma_\beta

Hence for M_{(\alpha\beta)(\gamma\kappa)},

 \displaystyle M_{(\alpha\beta)(\gamma\kappa)} = \frac{1}{4}{\rm tr}\left\{U(\sigma_\alpha\otimes\sigma_\beta)
U(\sigma_\gamma\otimes\sigma_\kappa)\right\}

  \displaystyle = \frac{1}{4}{\rm tr}\Big\{\left( \mid 0\rangle\langle 0\mid \sigma_\alpha\otimes\sigma_\beta + \mid 1\rangle\langle 1\mid\sigma_\alpha\otimes\sigma_x\sigma_\beta\right)

     \displaystyle \times\,\left( \mid 0\rangle\langle 0\mid \sigma_\gamma\otimes\sigma_\kappa + \mid 1\rangle\langle 1\mid\sigma_\gamma\otimes\sigma_x\sigma_\kappa\right)\Big\}

  \displaystyle = \frac{1}{4}\Big\{2(\sigma_\alpha)_{00}\delta_{\beta\kappa}+(\sigma_\alpha)_{01}(\sigma_\gamma)_{10}{\rm tr}(\sigma_\beta\sigma_x\sigma_\kappa)

      \displaystyle{}+(\sigma_\alpha)_{10}(\sigma_\gamma)_{01}{\rm tr}(\sigma_\kappa\sigma_x\sigma_\beta)+(\sigma_\alpha)_{11}(\sigma_\gamma)_{11}{\rm tr}(\sigma_x\sigma_\beta\sigma_x\sigma_\kappa)\Big\}

For \chi_{(\alpha\beta)(\gamma\kappa)},

 \displaystyle \chi_{(\alpha\beta)(\gamma\kappa)} = \frac{1}{16}{\rm tr}\left\{U(\sigma_\alpha\otimes\sigma_\beta)\right\}{\rm tr}\left\{U(\sigma_\gamma\otimes\sigma_\kappa)\right\}

  \displaystyle=\frac{1}{16}\left\{(\sigma_\alpha)_{00}\sigma_\beta+(\sigma_\alpha)_{11}\sigma_x\sigma_\beta\right\}
\left\{(\sigma_\gamma)_{00}\sigma_\kappa+(\sigma_\gamma)_{11}\sigma_x\sigma_\kappa\right\}

8.10

The error mapping process is described as:

 \displaystyle \epsilon(\rho) = \begin{pmatrix}
\rho_{11}e^{-\frac{t}{T_1}} & \rho_{10}e^{-i\Delta t-\frac{t}{T^*_2}} \\
\rho_{01}e^{i\Delta t-\frac{t}{T^*_2}}  & \rho_{00} + (1-e^{-\frac{t}{T_1}})\rho_{11} \end{pmatrix}

In the operator form, this can be described as:

 \displaystyle\epsilon\left(\mid 0\rangle\langle 0\mid\right) = \mid 0\rangle\langle 0 \mid

 \displaystyle\epsilon\left(\mid 1\rangle\langle 1\mid\right) = e^{-\frac{t}{T_1}}\mid 1\rangle\langle 1 \mid + \left(1-e^{-\frac{t}{T_1}}\right)\mid 0\rangle\langle 0 \mid

 \displaystyle\epsilon\left(\mid 0\rangle\langle 1\mid\right) = e^{i\Delta t-\frac{t}{T^*_2}}\mid 0\rangle\langle 1 \mid

 \displaystyle\epsilon\left(\mid 1\rangle\langle 0\mid\right) = e^{-i\Delta t-\frac{t}{T^*_2}}\mid 1\rangle\langle 0 \mid

Now the matrix elements of \chi are calculated through the relationships:

 \displaystyle\chi_{\alpha\beta} = \langle\alpha\mid(\mathbb 1\otimes\epsilon)(\mid\Psi\rangle\langle\Psi\mid)\mid\beta\rangle

  \displaystyle = \frac{1}{d}\sum_{i,j}\langle\alpha\mid i\rangle\langle j\mid \otimes\epsilon(\mid i\rangle\langle j\mid)\mid\beta\rangle

  \displaystyle = \frac{1}{d^2}\sum_{i,j}\sum_{k,l}\langle k\mid i\rangle\langle j\mid l\rangle\otimes\langle k\mid\hat S^\dagger_\alpha\epsilon(\mid i\rangle\langle j\mid)\hat S_\beta\mid l\rangle

  \displaystyle = \frac{1}{d^2}\sum_{i,j}\langle i\mid\hat S^\dagger_\alpha\epsilon(\mid i\rangle\langle j\mid)\hat S_\beta\mid j\rangle

For our particular case, using the notation \displaystyle \sigma_\alpha,\, \sigma_\beta \in \{\sigma_0 := \mathbb 1,\sigma_x,\sigma_y,\sigma_z\}:

 \displaystyle\chi_{\alpha\beta} = \frac{1}{4}\Big\{\langle 0\mid \sigma_\alpha\mid 0\rangle\langle 0\mid \sigma_\beta\mid 0\rangle

      \displaystyle{}+\langle 1\mid \sigma_\alpha\mid 1\rangle\langle 1\mid \sigma_\beta\mid 1\rangle e^{-\frac{t}{T_1}}+
\langle 1\mid \sigma_\alpha\mid 0\rangle\langle 0\mid \sigma_\beta\mid 1\rangle\left(1- e^{-\frac{t}{T_1}}\right)

      \displaystyle{}+\langle 0\mid \sigma_\alpha\mid 0\rangle\langle 1\mid \sigma_\beta\mid 1\rangle e^{i\Delta t-\frac{t}{T^*_2}}

      \displaystyle{}+\langle 1\mid \sigma_\alpha\mid 1\rangle\langle 0\mid \sigma_\beta\mid 0\rangle e^{-i\Delta t-\frac{t}{T^*_2}}\Big\}

Non-zero elements are:

 \displaystyle\chi_{00} = \frac{1}{4}\left\{1+e^{-\frac{t}{T_1}}+2e^{-\frac{t}{T^*_2}}\cos(\Delta t)\right\}

 \displaystyle\chi_{0z} = \frac{1}{4}\left\{-1+e^{-\frac{t}{T_1}}+2ie^{-\frac{t}{T^*_2}}\sin(\Delta t)\right\}

 \displaystyle\chi_{z0}  = \frac{1}{4}\left\{-1+e^{-\frac{t}{T_1}}-2ie^{-\frac{t}{T^*_2}}\sin(\Delta t)\right\}

 \displaystyle\chi_{zz} = \frac{1}{4}\left\{1+e^{-\frac{t}{T_1}}-2e^{-\frac{t}{T^*_2}}\cos(\Delta t)\right\}

 \displaystyle\chi_{xx} = \frac{1}{4}\left(1-e^{-\frac{t}{T_1}}\right)

 \displaystyle\chi_{xy} = -\chi_{yx} = \frac{i}{4}\left(1-e^{-\frac{t}{T_1}}\right)

 \displaystyle\chi_{yy} = \frac{1}{4}\left(1-e^{-\frac{t}{T_1}}\right)

 \displaystyle\therefore\,\chi_{\epsilon_1} = \frac{1}{4}\begin{pmatrix}
1+e^{-\frac{t}{T_1}}+2e^{-\frac{t}{T^*_2}}\cos(\Delta t) & -1+e^{-\frac{t}{T_1}}+2ie^{-\frac{t}{T^*_2}}\sin(\Delta t) & 0 & 0 \\ -1+e^{-\frac{t}{T_1}}-2ie^{-\frac{t}{T^*_2}}\sin(\Delta t) & 1+e^{-\frac{t}{T_1}}-2e^{-\frac{t}{T^*_2}}\cos(\Delta t) & 0 & 0 \\
0 & 0 & 1-e^{-\frac{t}{T_1}} & i \left(1-e^{-\frac{t}{T_1}}\right) \\
0 & 0 &  -i \left(1-e^{-\frac{t}{T_1}}\right) & 1-e^{-\frac{t}{T_1}}
\end{pmatrix}

By taking the limit T_1, T^*_2 \to \infty, the error-free version is:

 \displaystyle\therefore\,\chi_{\epsilon_0} = \frac{1}{4}\begin{pmatrix}
2+2\cos(\Delta t) & 2i\sin(\Delta t) & 0 & 0 \\ -2i\sin(\Delta t) & 2-2\cos(\Delta t) & 0 & 0 \\
0 & 0 & 0 & 0 \\
0 & 0 & 0 & 0
\end{pmatrix}

Hence, the process fidelity is:

 \displaystyle F_{\rm prc} = {\rm tr}\left(\chi_{\epsilon_0}^\dagger\chi_{\epsilon_1}\right)

  \displaystyle = \frac{1}{16}\Big\{2(1+\cos(\Delta t))\left(1+e^{-\frac{t}{T_1}}+2e^{-\frac{t}{T^*_2}}\cos(\Delta t)\right)

     \displaystyle {} + 2i\sin(\Delta t)\left(-1+e^{-\frac{t}{T_1}}-2ie^{-\frac{t}{T^*_2}}\sin(\Delta t)\right)

     \displaystyle {} + 2(1-\cos(\Delta t))\left(1+e^{-\frac{t}{T_1}}-2e^{-\frac{t}{T^*_2}}\cos(\Delta t)\right)

     \displaystyle {} - 2i\sin(\Delta t)\left(-1+e^{-\frac{t}{T_1}}+2ie^{-\frac{t}{T^*_2}}\sin(\Delta t)\right)\Big\}

  \displaystyle = \frac{1}{4}\left\{1+e^{-\frac{t}{T_1}}+2e^{-\frac{t}{T^*_2}}\cos^2(\Delta t)+2e^{-\frac{t}{T^*_2}}\sin^2(\Delta t)\right\}

  \displaystyle = \frac{1}{4}\left(1+e^{-\frac{t}{T_1}}+2e^{-\frac{t}{T^*_2}}\right)

The average fidelity is:

 \displaystyle F_{\rm avg} = \frac{2F_{\rm proc}+1}{2+1} = \frac{2}{3}F_{\rm proc}+\frac{1}{3}

The matrix elements of M can be calculated through the relationship:

 \displaystyle M_{\beta'\alpha'} = \frac{1}{d} \sum_{\alpha,\beta}\chi_{\alpha\beta}{\rm tr}(\hat S_{\beta'}\hat S_\alpha\hat S_{\alpha'}\hat S_\beta^\dagger)

8.12

Define p_d as a probability of depolarization, and the totally depolarization channel is:

 \displaystyle\epsilon(\rho) = (1-p_d)\rho + p_d\frac{\mathbb 1}{d}

Hence, for a pure state \displaystyle\mid\psi\rangle\langle\psi\mid

 \displaystyle \epsilon(\mid\psi\rangle\langle\psi\mid) = (1-p_d)\mid\psi\rangle\langle\psi\mid + p_d\frac{\mathbb 1}{d}

So the fidelity for this particular state is:

 \displaystyle\langle\psi\mid \epsilon(\mid\psi\rangle\langle\psi\mid)\mid\psi\rangle = (1-p_d)+\frac{p_d}{d}

Since this is independent of the state, the average over all pure states is given by:

 \displaystyle F_{\rm avg} = (1-p_d)+\frac{p_d}{d}

Note that the depolarization parameter p in the textbook (p.220) is a probability of not decaying: p = 1 - p_d. So (8.24) should be:

 \displaystyle F_{\rm avg} = (1-p_d)+\frac{p_d}{d} = p + \frac{1-p}{d}

Derivation of Input-Output relations for waveguide-QED

Target system

The system is similar to the following one.

enakai00.hatenablog.com

Here we restrict the states with only one excitation (either qubit or a propagating photon).

 \displaystyle \mid\psi(t)\rangle = c_1(t)\mid 1,{\rm vac}\rangle + \sum_k\psi_k(t)\mid 0,1_k\rangle --- (7.13)

The Heisenberg equations are:

 \displaystyle i\dot c_1(t) = \Delta c_1(t) + \sum_kg_k\psi_k(t),\ i\dot\psi_k(t) = \omega_k\psi_k(t) + g_k^* c_1(t) --- (7.14)

Using the same argument to derive (B.23) in the above article, we have the formal solution for \psi_k(t) as:

 \displaystyle\psi(t) = e^{-i\omega_k(t-t_0)}-ig_k^*\int_{t_0}^te^{-i\omega_k(t-\tau)}c(\tau)\,d\tau --- (1)

Position space

Wavefunction of the photon field is defined as:

 \displaystyle \Psi(x, t) := \frac{1}{\sqrt{d}}\langle x\mid\psi(t)\rangle =  \frac{1}{\sqrt{d}}\sum_k\psi_k(t)\langle x \mid 1_k\rangle =  \frac{1}{\sqrt{d}}\sum_k\psi_k(t)e^{ikx}

We included \displaystyle\frac{1}{\sqrt{d}} so that the wavefunction is scale-independent.

We split it into right-moving part \Psi_+(x, t) and left-moving part \Psi_-(x, t) as:

 \displaystyle \Psi_+(x, t) = \frac{1}{\sqrt{d}}\sum_{k > 0}\psi_k(t)e^{\pm ikx}

By substituting (1), we have:

 \displaystyle \Psi_\pm(x, t) = \Psi_\pm(x, t, t_0) -i \frac{1}{\sqrt{d}}\sum_{k > 0}g_k^*\int_{t_0}^t e^{\pm ikx-i\omega_k(t-\tau)}c_1(\tau)\,d\tau --- (7.17)

 \displaystyle \Psi_\pm(x,t,t_0) := \frac{1}{\sqrt{d}}\sum_{k>0}e^{\pm ikx-i\omega_k(t-t_0)}

where \displaystyle \Psi_\pm(x,t,t_0) represents a group of plane waves corresponds to input (t_0=-\infty) or output (t_0=\infty) photon beam. Note that in some locations, they are just formal expressions. For example, \Psi_+(x<0, t, \infty) corresponds to a right-moving output beam (before reaching the resonator) that doesn't exist in reality. It's a formal extension of the real output beam in x>0 to x<0.

Approximate calculation

We will apply approximation to the last term in (7.17) with the following assumptions:

 \displaystyle g_k = \frac{g}{\sqrt{d}},\ \omega = vk,\,\frac{2\pi}{d}\sum_{k>0} \sim \int_0^\infty \frac{dk}{d\omega}\,d\omega =  \frac{1}{v}\int_0^\infty \frac{dk}{d\omega}\,d\omega

The last one is the same as in this. Now,

 \displaystyle  -i \frac{1}{\sqrt{d}}\sum_{k > 0}g_k^*\int_{t_0}^t e^{\pm ikx-i\omega_k(t-\tau)}c_1(\tau)\,d\tau

  \displaystyle = -i\frac{g}{2\pi v} \int_{t_0}^t\left\{\int_0^\infty e^{\pm i\omega\frac{x}{v}-i\omega(t-\tau)}d\omega\right\}c_1(\tau)\,d\tau

  \displaystyle = -i\frac{g}{2\pi v} \int_{t_0}^t\left\{\int_0^\infty e^{i\omega(\tau-t\pm \frac{x}{v})}d\omega\right\}c_1(\tau)\,d\tau --- (2)

As the integration with \omega will be dominant around \displaystyle \tau \sim t\mp\frac{x}{v},

  • If the integral interval of \tau contains \displaystyle \tau \sim t\mp\frac{x}{v}, we replace c_1(\tau) with c_1\left(t\mp\frac{x}{v}\right), and extend the integral interval of \tau to -\infty to \infty.
  • If not, we approximate as (2) \sim 0.

The followings are the cases that result in (2) \sim 0. They corresponds to the following trivial cases.

  • \displaystyle \Psi_+(x<0, t) and t_0 = -\infty : In this case, \displaystyle \Psi_+(x<0, t) is an input beam before reaching to the resonator. So it's naturally the same as \displaystyle \Psi_+(x<0, t, -\infty).
  • \displaystyle \Psi_+(x>0, t) and t_0 = \infty : In this case, \displaystyle \Psi_+(x>0, t) is an output beam leaving the resonator. So it's naturally the same as \displaystyle \Psi_+(x>0, t, \infty).
  • \displaystyle \Psi_-(x>0, t) and t_0 = -\infty : In this case, \displaystyle \Psi_+(x>0, t) is an input beam before reaching to the resonator. So it's naturally the same as \displaystyle \Psi_-(x>0, t, -\infty).
  • \displaystyle \Psi_-(x<0, t) and t_0 = \infty : In this case, \displaystyle \Psi_+(x<0, t) is an output beam leaving the resonator. So it's naturally the same as \displaystyle \Psi_-(x<0, t, \infty).

Note: Strictly speaking, \displaystyle \Psi_+(x>0, t<\infty) is different from the asymptotic plane waves \displaystyle\Psi_+(x>0, t, t_0=\infty). In this rough approximation, we simply drop this information. Instead, we extract that information from \displaystyle \Psi_+(x>0, t<\infty) - \Psi_+(x>0, t, t_0=-\infty). In other words, we shouldn't use the trivial cases above to extract any information about the photon-resonator interaction.

In other cases, (2) represents the difference of the beam before and after interacting with the resonator. In the following calculation, we implicitly exclude the trivial cases above.

For the case t_0=-\infty,

 \displaystyle (2) = -i\frac{g}{2\pi v}c_1\left(t\mp\frac{x}{v}\right)\int_{-\infty}^\infty \int_0^\infty e^{i\omega(\tau-t\pm \frac{x}{v})}\,d\omega\,d\tau

  \displaystyle = -i\frac{g}{2\pi v}c_1\left(t\mp\frac{x}{v}\right)\int_{-\infty}^\infty\pi\delta\left(\tau-t\pm\frac{x}{v}\right) + i\,{\mathcal PV}\left(\frac{1}{\tau-t\pm\frac{x}{v}}\right)\,d\tau

  \displaystyle =-i\frac{g}{2 v}c_1\left(t\mp\frac{x}{v}\right)

For the case t_0=\infty,

 \displaystyle (2) = i\frac{g}{2\pi v}c_1\left(t\mp\frac{x}{v}\right)\int_{-\infty}^\infty \int_0^\infty e^{i\omega(\tau-t\pm \frac{x}{v})}\,d\omega\,d\tau

  \displaystyle = i\frac{g}{2\pi v}c_1\left(t\mp\frac{x}{v}\right)\int_{-\infty}^\infty\pi\delta\left(\tau-t\pm\frac{x}{v}\right) + i\,{\mathcal PV}\left(\frac{1}{\tau-t\pm\frac{x}{v}}\right)\,d\tau

  \displaystyle = i\frac{g}{2 v}c_1\left(t\mp\frac{x}{v}\right)

So we have the following non-trivial cases:

 \displaystyle\Psi_+(x>0,t) = \Psi^{\rm in}_+(x>0,t) - i\frac{g}{2v}c_1\left(t-\frac{x}{v}\right)

 \displaystyle\Psi_+(x<0,t) = \Psi^{\rm out}_+(x<0,t) + i\frac{g}{2v}c_1\left(t-\frac{x}{v}\right)

 \displaystyle\Psi_-(x<0,t) = \Psi^{\rm in}_+(x<0,t) - i\frac{g}{2v}c_1\left(t+\frac{x}{v}\right)

 \displaystyle\Psi_-(x>0,t) = \Psi^{\rm out}_-(x>0,t) + i\frac{g}{2v}c_1\left(t+\frac{x}{v}\right)

where \displaystyle\psi^{\rm in}_{\pm}(x, t) := \psi_{\pm}(x, t, -\infty),\, \psi^{\rm out}_{\pm}(x, t) := \psi_{\pm}(x, t, \infty)

Input-output relations

From the equation:

 \displaystyle \lim_{x\to 0} \Psi_+(x>0,t) = \lim_{x\to 0} \Psi_+(x<0,t)

 \displaystyle \lim_{x\to 0} \Psi_-(x>0,t) = \lim_{x\to 0} \Psi_-(x<0,t)

we have:

 \displaystyle\Psi^{\rm out}_\pm(0, t) = \Psi^{\rm in}_\pm(0,t) - i\frac{g}{v}c_1(t)

Using \displaystyle\gamma = \frac{2|g|^2}{v}, this can be written as:

 \displaystyle\Psi^{\rm out}_\pm(0, t) = \Psi^{\rm in}_\pm(0,t) - i\sqrt{\frac{\gamma}{2v}}c_1(t) --- (7.20)